Convergence in probability












4












$begingroup$


enter image description here



Can anyone tell me how they got the regions $0<epsilon<theta$ and $epsilon >0 $.



Also to clarify, is the last step where it says $lim_{n to infty} P(|Y_n-theta|>epsilon)=0$










share|cite|improve this question











$endgroup$

















    4












    $begingroup$


    enter image description here



    Can anyone tell me how they got the regions $0<epsilon<theta$ and $epsilon >0 $.



    Also to clarify, is the last step where it says $lim_{n to infty} P(|Y_n-theta|>epsilon)=0$










    share|cite|improve this question











    $endgroup$















      4












      4








      4





      $begingroup$


      enter image description here



      Can anyone tell me how they got the regions $0<epsilon<theta$ and $epsilon >0 $.



      Also to clarify, is the last step where it says $lim_{n to infty} P(|Y_n-theta|>epsilon)=0$










      share|cite|improve this question











      $endgroup$




      enter image description here



      Can anyone tell me how they got the regions $0<epsilon<theta$ and $epsilon >0 $.



      Also to clarify, is the last step where it says $lim_{n to infty} P(|Y_n-theta|>epsilon)=0$







      probability sequences-and-series convergence random-variables uniform-distribution






      share|cite|improve this question















      share|cite|improve this question













      share|cite|improve this question




      share|cite|improve this question








      edited Dec 17 '18 at 22:54









      Davide Giraudo

      127k17154268




      127k17154268










      asked Jun 5 '16 at 14:23









      HueHueHueHue

      12310




      12310






















          2 Answers
          2






          active

          oldest

          votes


















          3












          $begingroup$

          begin{align}
          mathbb{P}(|Y_n - theta | geq epsilon) &= 1- mathbb{P}(-epsilon leq Y_n - theta leq epsilon) \
          &= 1 - mathbb{P}(Y_n leq theta + epsilon) + mathbb{P}(Y_n leq theta - epsilon) \
          &= 1 - F_{Y_n}(theta + epsilon) + F_{Y_n}(theta-epsilon). \
          &= F_{Y_n}(theta- epsilon).
          end{align}

          Note that the last equality follows from the fact that $theta + epsilon geq theta$ for any $epsilon>0$ and consequently $F_{Y_n}(theta+ epsilon) = 1$. According to the definition of the distribution function of $Y_n$, there are two cases to be considered:



          (1) Assume $0<epsilon<theta$, then $0 < theta-epsilon < theta$ and consequently
          $$mathbb{P}(|Y_n - theta| geq epsilon) = left(frac{theta-epsilon}{theta}right)^n to 0,$$
          as $n to infty$.



          (2) Assume $epsilon > theta$, then $theta - epsilon < 0$ and consequently $F_{Y_n}(theta- epsilon) = 0, forall n geq 1$. Therefore, the result holds in the limit.






          share|cite|improve this answer











          $endgroup$













          • $begingroup$
            I'm a little confused with your statement, Shouldn't $F_{yn}(theta -epsilon)=(frac{theta -epsilon}{theta})^{n}$
            $endgroup$
            – HueHue
            Jun 5 '16 at 16:14








          • 1




            $begingroup$
            Yes ... and $mathbb{P}(|Y_n -theta | geq epsilon) = F_{Y_n}(theta-epsilon)$ as showed above.
            $endgroup$
            – Siron
            Jun 5 '16 at 16:26












          • $begingroup$
            Sorry I meant I was confused with the bit where you said "consider $epsilon geq theta$. In that case..."
            $endgroup$
            – HueHue
            Jun 5 '16 at 16:31










          • $begingroup$
            What exactly are you confused about?
            $endgroup$
            – Siron
            Jun 5 '16 at 16:45



















          -1












          $begingroup$

          Imagine we have $X$ representing the toss of a fair die.



          Then $text{Range}(X) = {1,2,cdots,6}$



          $$P(X = x) = frac16$$



          So:



          $$P(X le 1) = frac16$$
          $$P(X le 2) = frac26$$
          $$P(X le 3) = frac36$$
          $$P(X le 4) = frac46$$
          $$P(X le 5) = frac56$$
          $$P(X le 6) = frac66 = 1$$



          Also:



          $$P(X le 7) = frac66 = 1$$
          $$P(X le 0) = frac06$$
          $$P(X le 0.5) = frac06$$
          $$P(X le -2000) = frac06$$
          $$P(X le 4.5) = frac46$$
          $$P(X > 7) = frac06$$



          Thus:



          The '$varepsilon > theta$' (I'll assume '$varepsilon > 0$' is a typo) is like '$X > 7$'



          The '$0 < varepsilon < theta$' (I'll assume '$varepsilon > 0$' is a typo) is like '$1 < X < 6$'





          Also



          Show that $Y_n$ converges in probability






          share|cite|improve this answer











          $endgroup$













            Your Answer





            StackExchange.ifUsing("editor", function () {
            return StackExchange.using("mathjaxEditing", function () {
            StackExchange.MarkdownEditor.creationCallbacks.add(function (editor, postfix) {
            StackExchange.mathjaxEditing.prepareWmdForMathJax(editor, postfix, [["$", "$"], ["\\(","\\)"]]);
            });
            });
            }, "mathjax-editing");

            StackExchange.ready(function() {
            var channelOptions = {
            tags: "".split(" "),
            id: "69"
            };
            initTagRenderer("".split(" "), "".split(" "), channelOptions);

            StackExchange.using("externalEditor", function() {
            // Have to fire editor after snippets, if snippets enabled
            if (StackExchange.settings.snippets.snippetsEnabled) {
            StackExchange.using("snippets", function() {
            createEditor();
            });
            }
            else {
            createEditor();
            }
            });

            function createEditor() {
            StackExchange.prepareEditor({
            heartbeatType: 'answer',
            autoActivateHeartbeat: false,
            convertImagesToLinks: true,
            noModals: true,
            showLowRepImageUploadWarning: true,
            reputationToPostImages: 10,
            bindNavPrevention: true,
            postfix: "",
            imageUploader: {
            brandingHtml: "Powered by u003ca class="icon-imgur-white" href="https://imgur.com/"u003eu003c/au003e",
            contentPolicyHtml: "User contributions licensed under u003ca href="https://creativecommons.org/licenses/by-sa/3.0/"u003ecc by-sa 3.0 with attribution requiredu003c/au003e u003ca href="https://stackoverflow.com/legal/content-policy"u003e(content policy)u003c/au003e",
            allowUrls: true
            },
            noCode: true, onDemand: true,
            discardSelector: ".discard-answer"
            ,immediatelyShowMarkdownHelp:true
            });


            }
            });














            draft saved

            draft discarded


















            StackExchange.ready(
            function () {
            StackExchange.openid.initPostLogin('.new-post-login', 'https%3a%2f%2fmath.stackexchange.com%2fquestions%2f1814589%2fconvergence-in-probability%23new-answer', 'question_page');
            }
            );

            Post as a guest















            Required, but never shown

























            2 Answers
            2






            active

            oldest

            votes








            2 Answers
            2






            active

            oldest

            votes









            active

            oldest

            votes






            active

            oldest

            votes









            3












            $begingroup$

            begin{align}
            mathbb{P}(|Y_n - theta | geq epsilon) &= 1- mathbb{P}(-epsilon leq Y_n - theta leq epsilon) \
            &= 1 - mathbb{P}(Y_n leq theta + epsilon) + mathbb{P}(Y_n leq theta - epsilon) \
            &= 1 - F_{Y_n}(theta + epsilon) + F_{Y_n}(theta-epsilon). \
            &= F_{Y_n}(theta- epsilon).
            end{align}

            Note that the last equality follows from the fact that $theta + epsilon geq theta$ for any $epsilon>0$ and consequently $F_{Y_n}(theta+ epsilon) = 1$. According to the definition of the distribution function of $Y_n$, there are two cases to be considered:



            (1) Assume $0<epsilon<theta$, then $0 < theta-epsilon < theta$ and consequently
            $$mathbb{P}(|Y_n - theta| geq epsilon) = left(frac{theta-epsilon}{theta}right)^n to 0,$$
            as $n to infty$.



            (2) Assume $epsilon > theta$, then $theta - epsilon < 0$ and consequently $F_{Y_n}(theta- epsilon) = 0, forall n geq 1$. Therefore, the result holds in the limit.






            share|cite|improve this answer











            $endgroup$













            • $begingroup$
              I'm a little confused with your statement, Shouldn't $F_{yn}(theta -epsilon)=(frac{theta -epsilon}{theta})^{n}$
              $endgroup$
              – HueHue
              Jun 5 '16 at 16:14








            • 1




              $begingroup$
              Yes ... and $mathbb{P}(|Y_n -theta | geq epsilon) = F_{Y_n}(theta-epsilon)$ as showed above.
              $endgroup$
              – Siron
              Jun 5 '16 at 16:26












            • $begingroup$
              Sorry I meant I was confused with the bit where you said "consider $epsilon geq theta$. In that case..."
              $endgroup$
              – HueHue
              Jun 5 '16 at 16:31










            • $begingroup$
              What exactly are you confused about?
              $endgroup$
              – Siron
              Jun 5 '16 at 16:45
















            3












            $begingroup$

            begin{align}
            mathbb{P}(|Y_n - theta | geq epsilon) &= 1- mathbb{P}(-epsilon leq Y_n - theta leq epsilon) \
            &= 1 - mathbb{P}(Y_n leq theta + epsilon) + mathbb{P}(Y_n leq theta - epsilon) \
            &= 1 - F_{Y_n}(theta + epsilon) + F_{Y_n}(theta-epsilon). \
            &= F_{Y_n}(theta- epsilon).
            end{align}

            Note that the last equality follows from the fact that $theta + epsilon geq theta$ for any $epsilon>0$ and consequently $F_{Y_n}(theta+ epsilon) = 1$. According to the definition of the distribution function of $Y_n$, there are two cases to be considered:



            (1) Assume $0<epsilon<theta$, then $0 < theta-epsilon < theta$ and consequently
            $$mathbb{P}(|Y_n - theta| geq epsilon) = left(frac{theta-epsilon}{theta}right)^n to 0,$$
            as $n to infty$.



            (2) Assume $epsilon > theta$, then $theta - epsilon < 0$ and consequently $F_{Y_n}(theta- epsilon) = 0, forall n geq 1$. Therefore, the result holds in the limit.






            share|cite|improve this answer











            $endgroup$













            • $begingroup$
              I'm a little confused with your statement, Shouldn't $F_{yn}(theta -epsilon)=(frac{theta -epsilon}{theta})^{n}$
              $endgroup$
              – HueHue
              Jun 5 '16 at 16:14








            • 1




              $begingroup$
              Yes ... and $mathbb{P}(|Y_n -theta | geq epsilon) = F_{Y_n}(theta-epsilon)$ as showed above.
              $endgroup$
              – Siron
              Jun 5 '16 at 16:26












            • $begingroup$
              Sorry I meant I was confused with the bit where you said "consider $epsilon geq theta$. In that case..."
              $endgroup$
              – HueHue
              Jun 5 '16 at 16:31










            • $begingroup$
              What exactly are you confused about?
              $endgroup$
              – Siron
              Jun 5 '16 at 16:45














            3












            3








            3





            $begingroup$

            begin{align}
            mathbb{P}(|Y_n - theta | geq epsilon) &= 1- mathbb{P}(-epsilon leq Y_n - theta leq epsilon) \
            &= 1 - mathbb{P}(Y_n leq theta + epsilon) + mathbb{P}(Y_n leq theta - epsilon) \
            &= 1 - F_{Y_n}(theta + epsilon) + F_{Y_n}(theta-epsilon). \
            &= F_{Y_n}(theta- epsilon).
            end{align}

            Note that the last equality follows from the fact that $theta + epsilon geq theta$ for any $epsilon>0$ and consequently $F_{Y_n}(theta+ epsilon) = 1$. According to the definition of the distribution function of $Y_n$, there are two cases to be considered:



            (1) Assume $0<epsilon<theta$, then $0 < theta-epsilon < theta$ and consequently
            $$mathbb{P}(|Y_n - theta| geq epsilon) = left(frac{theta-epsilon}{theta}right)^n to 0,$$
            as $n to infty$.



            (2) Assume $epsilon > theta$, then $theta - epsilon < 0$ and consequently $F_{Y_n}(theta- epsilon) = 0, forall n geq 1$. Therefore, the result holds in the limit.






            share|cite|improve this answer











            $endgroup$



            begin{align}
            mathbb{P}(|Y_n - theta | geq epsilon) &= 1- mathbb{P}(-epsilon leq Y_n - theta leq epsilon) \
            &= 1 - mathbb{P}(Y_n leq theta + epsilon) + mathbb{P}(Y_n leq theta - epsilon) \
            &= 1 - F_{Y_n}(theta + epsilon) + F_{Y_n}(theta-epsilon). \
            &= F_{Y_n}(theta- epsilon).
            end{align}

            Note that the last equality follows from the fact that $theta + epsilon geq theta$ for any $epsilon>0$ and consequently $F_{Y_n}(theta+ epsilon) = 1$. According to the definition of the distribution function of $Y_n$, there are two cases to be considered:



            (1) Assume $0<epsilon<theta$, then $0 < theta-epsilon < theta$ and consequently
            $$mathbb{P}(|Y_n - theta| geq epsilon) = left(frac{theta-epsilon}{theta}right)^n to 0,$$
            as $n to infty$.



            (2) Assume $epsilon > theta$, then $theta - epsilon < 0$ and consequently $F_{Y_n}(theta- epsilon) = 0, forall n geq 1$. Therefore, the result holds in the limit.







            share|cite|improve this answer














            share|cite|improve this answer



            share|cite|improve this answer








            edited Dec 17 '18 at 22:18

























            answered Jun 5 '16 at 15:42









            SironSiron

            1,205614




            1,205614












            • $begingroup$
              I'm a little confused with your statement, Shouldn't $F_{yn}(theta -epsilon)=(frac{theta -epsilon}{theta})^{n}$
              $endgroup$
              – HueHue
              Jun 5 '16 at 16:14








            • 1




              $begingroup$
              Yes ... and $mathbb{P}(|Y_n -theta | geq epsilon) = F_{Y_n}(theta-epsilon)$ as showed above.
              $endgroup$
              – Siron
              Jun 5 '16 at 16:26












            • $begingroup$
              Sorry I meant I was confused with the bit where you said "consider $epsilon geq theta$. In that case..."
              $endgroup$
              – HueHue
              Jun 5 '16 at 16:31










            • $begingroup$
              What exactly are you confused about?
              $endgroup$
              – Siron
              Jun 5 '16 at 16:45


















            • $begingroup$
              I'm a little confused with your statement, Shouldn't $F_{yn}(theta -epsilon)=(frac{theta -epsilon}{theta})^{n}$
              $endgroup$
              – HueHue
              Jun 5 '16 at 16:14








            • 1




              $begingroup$
              Yes ... and $mathbb{P}(|Y_n -theta | geq epsilon) = F_{Y_n}(theta-epsilon)$ as showed above.
              $endgroup$
              – Siron
              Jun 5 '16 at 16:26












            • $begingroup$
              Sorry I meant I was confused with the bit where you said "consider $epsilon geq theta$. In that case..."
              $endgroup$
              – HueHue
              Jun 5 '16 at 16:31










            • $begingroup$
              What exactly are you confused about?
              $endgroup$
              – Siron
              Jun 5 '16 at 16:45
















            $begingroup$
            I'm a little confused with your statement, Shouldn't $F_{yn}(theta -epsilon)=(frac{theta -epsilon}{theta})^{n}$
            $endgroup$
            – HueHue
            Jun 5 '16 at 16:14






            $begingroup$
            I'm a little confused with your statement, Shouldn't $F_{yn}(theta -epsilon)=(frac{theta -epsilon}{theta})^{n}$
            $endgroup$
            – HueHue
            Jun 5 '16 at 16:14






            1




            1




            $begingroup$
            Yes ... and $mathbb{P}(|Y_n -theta | geq epsilon) = F_{Y_n}(theta-epsilon)$ as showed above.
            $endgroup$
            – Siron
            Jun 5 '16 at 16:26






            $begingroup$
            Yes ... and $mathbb{P}(|Y_n -theta | geq epsilon) = F_{Y_n}(theta-epsilon)$ as showed above.
            $endgroup$
            – Siron
            Jun 5 '16 at 16:26














            $begingroup$
            Sorry I meant I was confused with the bit where you said "consider $epsilon geq theta$. In that case..."
            $endgroup$
            – HueHue
            Jun 5 '16 at 16:31




            $begingroup$
            Sorry I meant I was confused with the bit where you said "consider $epsilon geq theta$. In that case..."
            $endgroup$
            – HueHue
            Jun 5 '16 at 16:31












            $begingroup$
            What exactly are you confused about?
            $endgroup$
            – Siron
            Jun 5 '16 at 16:45




            $begingroup$
            What exactly are you confused about?
            $endgroup$
            – Siron
            Jun 5 '16 at 16:45











            -1












            $begingroup$

            Imagine we have $X$ representing the toss of a fair die.



            Then $text{Range}(X) = {1,2,cdots,6}$



            $$P(X = x) = frac16$$



            So:



            $$P(X le 1) = frac16$$
            $$P(X le 2) = frac26$$
            $$P(X le 3) = frac36$$
            $$P(X le 4) = frac46$$
            $$P(X le 5) = frac56$$
            $$P(X le 6) = frac66 = 1$$



            Also:



            $$P(X le 7) = frac66 = 1$$
            $$P(X le 0) = frac06$$
            $$P(X le 0.5) = frac06$$
            $$P(X le -2000) = frac06$$
            $$P(X le 4.5) = frac46$$
            $$P(X > 7) = frac06$$



            Thus:



            The '$varepsilon > theta$' (I'll assume '$varepsilon > 0$' is a typo) is like '$X > 7$'



            The '$0 < varepsilon < theta$' (I'll assume '$varepsilon > 0$' is a typo) is like '$1 < X < 6$'





            Also



            Show that $Y_n$ converges in probability






            share|cite|improve this answer











            $endgroup$


















              -1












              $begingroup$

              Imagine we have $X$ representing the toss of a fair die.



              Then $text{Range}(X) = {1,2,cdots,6}$



              $$P(X = x) = frac16$$



              So:



              $$P(X le 1) = frac16$$
              $$P(X le 2) = frac26$$
              $$P(X le 3) = frac36$$
              $$P(X le 4) = frac46$$
              $$P(X le 5) = frac56$$
              $$P(X le 6) = frac66 = 1$$



              Also:



              $$P(X le 7) = frac66 = 1$$
              $$P(X le 0) = frac06$$
              $$P(X le 0.5) = frac06$$
              $$P(X le -2000) = frac06$$
              $$P(X le 4.5) = frac46$$
              $$P(X > 7) = frac06$$



              Thus:



              The '$varepsilon > theta$' (I'll assume '$varepsilon > 0$' is a typo) is like '$X > 7$'



              The '$0 < varepsilon < theta$' (I'll assume '$varepsilon > 0$' is a typo) is like '$1 < X < 6$'





              Also



              Show that $Y_n$ converges in probability






              share|cite|improve this answer











              $endgroup$
















                -1












                -1








                -1





                $begingroup$

                Imagine we have $X$ representing the toss of a fair die.



                Then $text{Range}(X) = {1,2,cdots,6}$



                $$P(X = x) = frac16$$



                So:



                $$P(X le 1) = frac16$$
                $$P(X le 2) = frac26$$
                $$P(X le 3) = frac36$$
                $$P(X le 4) = frac46$$
                $$P(X le 5) = frac56$$
                $$P(X le 6) = frac66 = 1$$



                Also:



                $$P(X le 7) = frac66 = 1$$
                $$P(X le 0) = frac06$$
                $$P(X le 0.5) = frac06$$
                $$P(X le -2000) = frac06$$
                $$P(X le 4.5) = frac46$$
                $$P(X > 7) = frac06$$



                Thus:



                The '$varepsilon > theta$' (I'll assume '$varepsilon > 0$' is a typo) is like '$X > 7$'



                The '$0 < varepsilon < theta$' (I'll assume '$varepsilon > 0$' is a typo) is like '$1 < X < 6$'





                Also



                Show that $Y_n$ converges in probability






                share|cite|improve this answer











                $endgroup$



                Imagine we have $X$ representing the toss of a fair die.



                Then $text{Range}(X) = {1,2,cdots,6}$



                $$P(X = x) = frac16$$



                So:



                $$P(X le 1) = frac16$$
                $$P(X le 2) = frac26$$
                $$P(X le 3) = frac36$$
                $$P(X le 4) = frac46$$
                $$P(X le 5) = frac56$$
                $$P(X le 6) = frac66 = 1$$



                Also:



                $$P(X le 7) = frac66 = 1$$
                $$P(X le 0) = frac06$$
                $$P(X le 0.5) = frac06$$
                $$P(X le -2000) = frac06$$
                $$P(X le 4.5) = frac46$$
                $$P(X > 7) = frac06$$



                Thus:



                The '$varepsilon > theta$' (I'll assume '$varepsilon > 0$' is a typo) is like '$X > 7$'



                The '$0 < varepsilon < theta$' (I'll assume '$varepsilon > 0$' is a typo) is like '$1 < X < 6$'





                Also



                Show that $Y_n$ converges in probability







                share|cite|improve this answer














                share|cite|improve this answer



                share|cite|improve this answer








                edited Apr 13 '17 at 12:19









                Community

                1




                1










                answered Nov 2 '16 at 15:04









                BCLCBCLC

                1




                1






























                    draft saved

                    draft discarded




















































                    Thanks for contributing an answer to Mathematics Stack Exchange!


                    • Please be sure to answer the question. Provide details and share your research!

                    But avoid



                    • Asking for help, clarification, or responding to other answers.

                    • Making statements based on opinion; back them up with references or personal experience.


                    Use MathJax to format equations. MathJax reference.


                    To learn more, see our tips on writing great answers.




                    draft saved


                    draft discarded














                    StackExchange.ready(
                    function () {
                    StackExchange.openid.initPostLogin('.new-post-login', 'https%3a%2f%2fmath.stackexchange.com%2fquestions%2f1814589%2fconvergence-in-probability%23new-answer', 'question_page');
                    }
                    );

                    Post as a guest















                    Required, but never shown





















































                    Required, but never shown














                    Required, but never shown












                    Required, but never shown







                    Required, but never shown

































                    Required, but never shown














                    Required, but never shown












                    Required, but never shown







                    Required, but never shown







                    Popular posts from this blog

                    Plaza Victoria

                    In PowerPoint, is there a keyboard shortcut for bulleted / numbered list?

                    How to put 3 figures in Latex with 2 figures side by side and 1 below these side by side images but in...